Mathcenter Forum

Mathcenter Forum (https://www.mathcenter.net/forum/index.php)
-   พีชคณิต (https://www.mathcenter.net/forum/forumdisplay.php?f=15)
-   -   พหุนามครับ (https://www.mathcenter.net/forum/showthread.php?t=23212)

RER 05 เมษายน 2016 13:22

พหุนามครับ
 
กำหนดให้ $c$ เป็นจำนวนตรรกยะ จงแสดงว่า
$x^3-3cx^2-3cx+c=0$
มีรากตรรกยะได้อย่างมาก 1 ตัว ช่วยหน่อยครับ:please:

Thgx0312555 05 เมษายน 2016 22:53

สมมติ $\dfrac{p_1}{q_1},\dfrac{p_2}{q_2}$ เป็นคำตอบที่แตกต่างกันของสมการนี้

โดยที่ $p_1,q_1,p_2,q_2 \in \mathbb{Z}, \gcd(p_1,q_1)=\gcd(p_2,q_2)=1, p_1,p_2 \ge 0$

สมการดังกล่าวสามารถจัดรูปใหม่ได้เป็น $c=\dfrac{x^3}{3x^2+3x-1}$

ดังนั้นจะได้ว่า $c=\dfrac{p_1^3}{3p_1^2q_1+3p_1q_1^2-q_1^3}=\dfrac{p_2^3}{3p_2^2q_2+3p_2q_2^2-q_2^3} \quad (\ast )$

จาก $\gcd(p_1^3,3p_1^2q_1+3p_1q_1^2-q_1^3)=\gcd(p_2^3,3p_2^2q_2+3p_2q_2^2-q_2^3)=1, p_1,p_2\ge 0$

จะได้ $p_1=p_2$
กรณี $p_1=p_2=0$ เห็นได้ชัดว่าจริงจึงสมมติ $p=p_1=p_2 \neq 0$

ดังนั้น $(\ast )$ จะเขียนใหม่เป็น $3p^2q_1+3pq_1^2-q_1^3=3p^2q_2+3pq_2^2-q_2^3$

$3p^2(q_1-q_2)+3p(q_1^2-q_2^2)=q_1^3-q_2^3$

จาก $q_1 \neq q_2$, หารด้วย $q_1-q_2$ ตลอด
$3p^2+3pq_1+3pq_2=q_1^2+q_1q_2+q_2^2$

ต่อมาจะแสดงว่าสมการ
$3x^2+3xy+3xz=y^2+yz+z^2$ มีคำตอบเดียวในจำนวนเต็มคือ $(0,0,0)$
สมมติว่ามีคำตอบอื่นนอกจาก $(0,0,0)$

ให้ $(A,B,C) \neq (0,0,0)$ เป็นคำตอบที่ทำให้ $|A|+|B|+|C|$ มีค่าน้อยที่สุด
$3A^2+3AB+3AC=B^2+BC+C^2 \quad ( \ast \ast )$
$3(A+B)(A+C)=B^2+4BC+C^2$

ถ้า $B \not\equiv C \pmod 2$ จะพบว่า $3(A+B)(A+C)$ เป็นคู่ แต่ $B^2+4BC+C^2$ เป็นคี่
ดังนั้น $B \equiv C \pmod 2$

ถ้า $B \equiv C \not\equiv A \pmod 2$ จะพบว่า $3(A+B)(A+C)$ เป็นคี่ แต่ $B^2+4BC+C^2$ เป็นคู่
ดังนั้น $B \equiv C \equiv A \pmod 2$

ถ้า $B \equiv C \equiv A \equiv 1 \pmod 2$ จะพบว่า $4 \mid 3(A+B)(A+C)$ แต่ $B^2+4BC+C^2 \equiv 2 \pmod 4$
ดังนั้น $B \equiv C \equiv A \equiv 0 \pmod 2$

นั่นคือ $2 \mid A, 2 \mid B, 2 \mid C$
แต่ถ้าจัดรูปสมการ $ ( \ast \ast )$ ใหม่จะได้ $3(\frac{A}{2})^2+3(\frac{A}{2}
)(\frac{B}{2})+3(\frac{A}{2})(\frac{C}{2})=(\frac{B}{2})^2+(\frac{B}{2})(\frac{C}{2})+(\frac{C}{2})^2$

นั่นคือ $(\frac{A}{2},\frac{B}{2},\frac{C}{2})$ เป็นคำตอบของสมการและ $|\frac{A}{2}|+|\frac{B}{2}|+|\frac{C}{2}|<|A|+|B|+|C|$
เกิดข้อขัดแย้ง

ดังนั้นสมการนี้ $3p^2+3pq_1+3pq_2=q_1^2+q_1q_2+q_2^2$ มีคำตอบเดียวคือ $p=q_1=q_2=0$
เกิดข้อขัดแย้ง

RER 06 เมษายน 2016 10:44

ขอบคุณครับ :)


เวลาที่แสดงทั้งหมด เป็นเวลาที่ประเทศไทย (GMT +7) ขณะนี้เป็นเวลา 07:58

Powered by vBulletin® Copyright ©2000 - 2024, Jelsoft Enterprises Ltd.
Modified by Jetsada Karnpracha